LSAT and Law School Admissions Forum

Get expert LSAT preparation and law school admissions advice from PowerScore Test Preparation.

 LustingFor!L
  • Posts: 80
  • Joined: Aug 27, 2016
|
#34312
In the example used to show when you have condition reasoning the stimulus and a weaken question (business firms retaining power and social responsibility), would answer choice E be how you would attack the question if it were causality? Showing effect without cause?
User avatar
 Dave Killoran
PowerScore Staff
  • PowerScore Staff
  • Posts: 5852
  • Joined: Mar 25, 2011
|
#34349
Hey Lust,

Yes, if you reframed this argument entirely, then the angle you are talking about would make sense. Basically, if you turned this into a conclusion where "Acting responsibility causes the retention of power," then (E) would provide examples where the cause occurred but the effect did not. To make that work would require some additional changes, mainly because the conclusion simply says "as long as it can," and there may be reasons beyond their control that ends power. So, they'd like eliminate outcomes like that in the stimulus so as to avoid the author saying that the company had in fact retained power the maximum possible time. But, if we reworked it a bit and cleaned it up, your idea would work. Good job!

Get the most out of your LSAT Prep Plus subscription.

Analyze and track your performance with our Testing and Analytics Package.